Ulikhetmaraton

Her kan brukere av forum utfordre hverandre med morsomme oppgaver og nøtter man ønsker å dele med andre. Dette er altså ikke et sted for desperate skrik om hjelp, de kan man poste i de andre forumene, men et sted for problemløsing på tvers av trinn og fag.

Moderatorer: Vektormannen, espen180, Aleks855, Solar Plexsus, Gustav, Nebuchadnezzar, Janhaa

mingjun
Cayley
Cayley
Innlegg: 91
Registrert: 18/11-2016 21:13
Sted: Det projektive planet

zzzivert skrev:Oppfølgere:
Vanskelig:
La [tex]x,y\in \mathbb{R}[/tex] slik at
[tex]x^3+y^3+\frac{x+y}{4}=\frac{15}{2}[/tex].
Vis at [tex]x+y\le 3[/tex].
$$\dfrac{15}{2}=x^3+y^3+\dfrac{x+y}{4}\geq 2\left(\dfrac{x+y}{2}\right)^3+\dfrac{x+y}{4}.$$

La $s=x+y$. Da har vi

\[2\left(\dfrac{s}{2}\right)^3+\dfrac{s}{4}\leq \dfrac{15}{2} \Leftrightarrow s^3+s-30\leq 0.\]

Det er lett å verifisere at venstre side av ulikheten, en tredjegradspolynom i $s$, er strengt stigende for alle $s$. Dermed oppfylles ulikheten for alle $s\leq r$, hvor $r^3+r-30=0$. $r=3$ er løsningen, og vi er ferdige.
mingjun
Cayley
Cayley
Innlegg: 91
Registrert: 18/11-2016 21:13
Sted: Det projektive planet

Vanskelig oppfølger:

La $x,y,z$ være positive reele tall. Bevis at \[ \left( xy+yz+zx \right)\left( \frac{1}{(x+y)^2} + \frac{1}{(y+z)^2} + \frac{1}{(z+x)^2} \right) \ge \frac{9}{4}.\]
Markus
Fermat
Fermat
Innlegg: 767
Registrert: 20/09-2016 13:48
Sted: NTNU

zzzivert skrev:Oppfølgere:
Middels:
La [tex]a,b,c\ge 0[/tex] slik at
[tex](a+1)(b+1)(c+1)=8[/tex].
Vis at [tex]abc\le 1[/tex].
Observer at $(a+1)(b+1)(c+1)=abc + ab+bc+ac+a+b+c+1$. AM-GM gir nå at $$\frac{abc+ab+bc+ac+a+b+c+1}{8} = 1 \geq \sqrt[8]{a^4b^4c^4} = \sqrt{abc} \\ \therefore abc \leq 1$$ Som skulle vises.

Vi har jo en til (vanskelig) oppfølger fra mingjun der oppe, så her er i tillegg en litt lettere en;
Gitt $a,b,c \in \mathbb{R}^+$, vis at $$\left(a+ \frac1b \right) \left(b + \frac1c \right) \left ( c + \frac1a \right) \geq 8$$
Gustav
Tyrann
Tyrann
Innlegg: 4555
Registrert: 12/12-2008 12:44

Markus skrev: Gitt $a,b,c \in \mathbb{R}^+$, vis at $$\left(a+ \frac1b \right) \left(b + \frac1c \right) \left ( c + \frac1a \right) \geq 8$$
AM-GM på hver faktor
Markus
Fermat
Fermat
Innlegg: 767
Registrert: 20/09-2016 13:48
Sted: NTNU

Ble litt dødt her, så her er en ganske lett ulikhet for å sette i gang tråden igjen.

Vis at $\tan(x) > x \enspace \forall x \in \left(0, \frac{\pi}{2} \right)$
Gustav
Tyrann
Tyrann
Innlegg: 4555
Registrert: 12/12-2008 12:44

Markus skrev:Ble litt dødt her, så her er en ganske lett ulikhet for å sette i gang tråden igjen.

Vis at $\tan(x) > x \enspace \forall x \in \left(0, \frac{\pi}{2} \right)$
Definer $f(x)=\tan x - x$ på $(0,\frac{\pi}{2})$, som er deriverbar med derivert $f'(x)=\tan^2 x>0$. Da gir sekantsetningen at $f(x)>0$
for alle $x\in (0,\frac{\pi}{2})$: Bevis ved motsigelse: Anta det fins en $c\in (0,\frac{\pi}{2})$ slik at $f(c)\le 0$. Da må det av sekantsetningen finnes en $d\in (0,c)$ slik at $f'(d)=\frac{f(c)-f(0)}{c-0}=\frac{f(c)}{c}\le 0$, som gir motsigelsen.

Oppfølger: La $a,b,c$ være positive reelle tall slik at $ab+bc+ca+abc=4$. Vis at $a+b+c\ge 3$

PS; beklager dersom denne er postet tidligere.
mingjun
Cayley
Cayley
Innlegg: 91
Registrert: 18/11-2016 21:13
Sted: Det projektive planet

Oppfølger: La $a,b,c$ være positive reelle tall slik at $ab+bc+ca+abc=4$. Vis at $a+b+c\ge 3$
Vi beviser dette kontrapositivt. Anta at $a+b+c<3$, da har vi $ab+bc+ca\leq\frac{\left(a+b+c\right)^2}{3}<\frac{3^2}{3}=3$, og $abc\leq\left(\dfrac{a+b+c}{3}\right)^3<1$. Dermed er $ab+bc+ca+abc<3+1=4$, og vi er ferdige.

Oppfølger: La $P(x)$ være et polynom med reele positive koeffisienter. Bevis at dersom $P(\frac{1}{x})\geq\frac{1}{P(x)}$ gjelder for $x=1$, så gjelder den for alle positive $x$.
Gustav
Tyrann
Tyrann
Innlegg: 4555
Registrert: 12/12-2008 12:44

mingjun skrev:
Oppfølger: La $a,b,c$ være positive reelle tall slik at $ab+bc+ca+abc=4$. Vis at $a+b+c\ge 3$
Vi beviser dette kontrapositivt. Anta at $a+b+c<3$, da har vi $ab+bc+ca\leq\frac{\left(a+b+c\right)^2}{3}<\frac{3^2}{3}=3$, og $abc\leq\left(\dfrac{a+b+c}{3}\right)^3<1$. Dermed er $ab+bc+ca+abc<3+1=4$, og vi er ferdige.
Kløktig løsning :D
Markus
Fermat
Fermat
Innlegg: 767
Registrert: 20/09-2016 13:48
Sted: NTNU

mingjun skrev:Oppfølger: La $P(x)$ være et polynom med reele positive koeffisienter. Bevis at dersom $P(\frac{1}{x})\geq\frac{1}{P(x)}$ gjelder for $x=1$, så gjelder den for alle positive $x$.
Denne har vært tidligere i tråden, så jeg tar meg den friheten å komme meg oppfølger. La $x,y,z \in \mathbb{R}^+$. Vis at $$\sqrt{x(3x+y)} + \sqrt{y(3y+z)} + \sqrt{z(3z+x)} \leq 2(x+y+z)$$
Gustav
Tyrann
Tyrann
Innlegg: 4555
Registrert: 12/12-2008 12:44

Cauchy-Schwarz gir at $\sqrt{x(3x+y)} + \sqrt{y(3y+z)} + \sqrt{z(3z+x)} \leq \sqrt{(x+y+z)(3x+y+3y+z+3z+x)}=2(x+y+z)$
Gustav
Tyrann
Tyrann
Innlegg: 4555
Registrert: 12/12-2008 12:44

Oppfølger: For positive reelle tall $x,y,z$, vis at $3^6\le (\frac{1}{x}+\frac{1}{y}+\frac{1}{z})^5(x^5+y^5+z^5)$.
Mattebruker

AM-GM-ulikskapen gir

( 1 ) ( [tex]\frac{1}{x}[/tex] + [tex]\frac{1}{y}[/tex] + [tex]\frac{1}{z}[/tex])[tex]^{5}[/tex] [tex]\geqslant[/tex](xyz)[tex]^{-\frac{5}{3}}[/tex][tex]\cdot[/tex]3[tex]^{5}[/tex]


( 2 ) (x[tex]^{5}[/tex] + y[tex]^{5}[/tex] + z[tex]^{5}[/tex] ) [tex]\geqslant[/tex]3[tex]\cdot[/tex](xyz)[tex]^{\frac{5}{3}}[/tex]


Multipliserer ( 1 ) med ( 2 ) og får den ulikskapen vi skulle vise.
Gustav
Tyrann
Tyrann
Innlegg: 4555
Registrert: 12/12-2008 12:44

Oppfølger: For $n\ge 2$, la $a_1,a_2,\cdots,a_n$ være positive reelle tall slik at $(a_1+a_2+\cdots + a_n)(\frac{1}{a_1}+\frac{1}{a_2}+\cdots + \frac{1}{a_n})\le (n+\frac12)^2$.

Vis at $\max (a_1,a_2,\cdots,a_n)\le 4 \min (a_1,a_2,\cdots, a_n)$.
zzzivert
Noether
Noether
Innlegg: 48
Registrert: 27/10-2014 09:26

Av symmetri kan vi anta at [tex]a_1\le a_2\le ... \le a_n[/tex]. Av homogenitet kan vi også anta at [tex]a_1=\frac{1}{2}[/tex]. La $x=a_n$.
Fra AM-GM har vi at
[tex]a_2+a_3+\cdots +a_{n-1} \ge (n-2) \sqrt[n-2]{a_2a_3 ... a_{n-1}}[/tex]
[tex]\frac{1}{a_2} + \frac{1}{a_3}+\cdots + \frac{1}{a_{n-1}}\ge \frac{n-2}{\sqrt[n-2]{a_2a_3 ... a_{n-1}}}[/tex]
La [tex]a=\sqrt[n-2]{a_2a_3 ... a_{n-1}}[/tex], da får vi ulikheten
[tex](a_1+a_2+\cdots +a_{n})(\frac{1}{a_1} + \frac{1}{a_2}+\cdots + \frac{1}{a_{n}})[/tex]
[tex]\ge (\frac{1}{2}+(n-2)a+x)(2+\frac{n-2}{a}+\frac{1}{x})[/tex]
[tex]= (\frac{2x+1}{2}+(n-2)a)(\frac{2x+1}{x}+\frac{n-2}{a})[/tex]
[tex]=\frac{(2x+1)^2}{2x}+(n-2)(\frac{2x+1}{2}\cdot \frac{1}{a}+\frac{2x+1}{x}\cdot a)+(n-2)^2[/tex]
[tex]\ge \frac{(2x+1)^2}{2x}+2(n-2)\sqrt{\frac{(2x+1)^2}{2x}}+(n-2)^2[/tex]

Anta nå at $x>2$. Da har vi også at
[tex](x-2)(8x-1)>0[/tex]
[tex]8x^2-17x+2>0[/tex]
[tex]4x^2+4x+1>\frac{25}{2}x[/tex]
[tex]\frac{(2x+1)^2}{2x}>\frac{25}{4}[/tex]

Defor får vi
[tex]\frac{(2x+1)^2}{2x}+2(n-2)\sqrt{\frac{(2x+1)^2}{2x}}+(n-2)^2>\frac{25}{4}+2(n-1)\frac{5}{2}+(n-2)^2=(n+\frac{1}{2})^2[/tex]
som er en motsigelse. Altså må [tex]x\le 2[/tex], som er ekvivalent med spørsmålet.
Markus
Fermat
Fermat
Innlegg: 767
Registrert: 20/09-2016 13:48
Sted: NTNU

Jeg mistenker at problemet, i likhet med mye annet, kan også løses på en annen måte. Med for eksempel Cauchy-Schwarz kan vi også finne en nedre grense for uttrykket; $(n+\frac12)^2 \geq \left (\sum_{i=1}^n a_i \right) \left ( \sum_{i=1}^n \frac{1}{a_i} \right) \geq n^2$ Den øvre og nedre grensen er så nærme hverandre, at jeg nesten tror det er med vilje, men jeg ser ikke helt veien videre herifra. Hvis noen vil hjelpe til eller gi hint settes det stor pris på!

Forresten, zzzivert, hvorfor kan du av homogenitet anta at $a_1=\frac12$?
Svar